آيا شما مايل به راه افتادن تاپيك ماراتن نامساوي مي باشي


  • مجموع رای دهندگان
    339

REZA 73

Active Member
ارسال ها
139
لایک ها
184
امتیاز
43
پاسخ : ماراتن نامساوي

من فرض خاصی نکردم ولی چیزی که مشهوده اگه a بزرگتر از یک باشه اون نامساوی درسته، البته میشه با روش لاگرانژ درستی این نابرابری جزئی رو به راحتی چک کرد، من با لاگرانژ بررسی کردم، ولی به احتمال زیاد یه اثبات خیلی ساده تر داره، متاسفانه به دلیل درس های زیاد دانشگاه فرصت نمیشه بیشتر از این رو مسایل فک کنم .
اصلا راه حل خودتون چیه؟سوال کجا بوده ؟
 

Dadgarnia

New Member
ارسال ها
1,350
لایک ها
1,127
امتیاز
0
پاسخ : ماراتن نامساوي

اصلا راه حل خودتون چیه؟سوال کجا بوده ؟
سوال كره بوده: AoPS Forum - Maximum with condition a^2 + b^2 + c^2 = 2abc + 1 • Art of Problem Solving راه حل منم خيلي شبيه به يكي از راه حل هايي كه اونجا نوشته هست ولي اگه بخواين مي تونم اينجا بنويسم. اگه شما موفق به كامل كردن راه حلتون شدين (بدون استفاده از لاگرانژ لطفا) همينجا بنويسين.
سوال بعد:
براي اعداد مثبت
رابطه ي
برقرار است. ثابت كنيد:

 

REZA 73

Active Member
ارسال ها
139
لایک ها
184
امتیاز
43
پاسخ : ماراتن نامساوي

سوال كره بوده: AoPS Forum - Maximum with condition a^2 + b^2 + c^2 = 2abc + 1 • Art of Problem Solving راه حل منم خيلي شبيه به يكي از راه حل هايي كه اونجا نوشته هست ولي اگه بخواين مي تونم اينجا بنويسم. اگه شما موفق به كامل كردن راه حلتون شدين (بدون استفاده از لاگرانژ لطفا) همينجا بنويسين.
سوال بعد:
براي اعداد مثبت
رابطه ي
برقرار است. ثابت كنيد:

با استفاده از لاگرانٰژ و تبدیل شرط به :




حالا اگه پرانتز ها رو از هم کم کنیم به راحتی نتیجه میشه a=b=c=dیعنی هر کدوم میشه:

با جایگذاری در معادله به راحتی حالت مینیمم به دست می آید و البته از
هم استفاده میشه که خیلی سادست.
 
آخرین ویرایش توسط مدیر

Dadgarnia

New Member
ارسال ها
1,350
لایک ها
1,127
امتیاز
0
پاسخ : ماراتن نامساوي

با استفاده از لاگرانٰژ و تبدیل شرط به :




حالا اگه پرانتز ها رو از هم کم کنیم به راحتی نتیجه میشه a=b=c=dیعنی هر کدوم میشه:

با جایگذاری در معادله به راحتی حالت مینیمم به دست می آید و البته از
هم استفاده میشه که خیلی سادست.
من خيلي در مورد لاگرانژ نمي دونم ولي فكر كنم درستش اينجوري باشه:
 

REZA 73

Active Member
ارسال ها
139
لایک ها
184
امتیاز
43
پاسخ : ماراتن نامساوي

من خيلي در مورد لاگرانژ نمي دونم ولي فكر كنم درستش اينجوري باشه:
فرقی نمیکنه اون یه ضریبه، در کل در روش لاگرانژ گرادیان ها موازی فرض میشن.:3:
 

Dadgarnia

New Member
ارسال ها
1,350
لایک ها
1,127
امتیاز
0
پاسخ : ماراتن نامساوي

سوال بعد:
برای اعداد مثبت
با شرط
ثابت کنید:

 
ارسال ها
6
لایک ها
4
امتیاز
0
پاسخ : ماراتن نامساوي

سوال بعد:
برای اعداد مثبت
با شرط
ثابت کنید:


نامساوی هم ارز است با:


روی هر ترم سمت چپ کوشی می زنیم:

و مساله حل می شود
 
ارسال ها
6
لایک ها
4
امتیاز
0
پاسخ : ماراتن نامساوي

فرض کنید
اعدادی حقیقی و مثبت هستند. نشان دهید اعداد
وجود دارند که

 

Dadgarnia

New Member
ارسال ها
1,350
لایک ها
1,127
امتیاز
0
پاسخ : ماراتن نامساوي

فرض کنید
اعدادی حقیقی و مثبت هستند. نشان دهید اعداد
وجود دارند که

فرض مي كنيم
باشه ضرايب رو قرار ميديم:
ابتدا فرض مي كنيم
فرد باشه. نامساوي هم ارزه با:


كه درستي اين نامساوي با توجه به فرضي كه كرديم بديهيه. حالا اگه
زوج باشه با توجه به درستي نامساوي در حالت قبل داريم:
كه درستي اين نامساوي هم بديهيه.

---- دو نوشته به هم متصل شده است ----

سوال بعد:
اگر
و
ثابت كنيد:

 

REZA 73

Active Member
ارسال ها
139
لایک ها
184
امتیاز
43
پاسخ : ماراتن نامساوي

فرض مي كنيم
باشه ضرايب رو قرار ميديم:
ابتدا فرض مي كنيم
فرد باشه. نامساوي هم ارزه با:


كه درستي اين نامساوي با توجه به فرضي كه كرديم بديهيه. حالا اگه
زوج باشه با توجه به درستي نامساوي در حالت قبل داريم:
كه درستي اين نامساوي هم بديهيه.

---- دو نوشته به هم متصل شده است ----

سوال بعد:
اگر
و
ثابت كنيد:

کلا چیزی که واضحه نامساوی همگنه پس فرض میکنیم :x+y+z=1
حالا اگه تابع f رو برابر((x/(x+k(1-x تعریف کنیم به راحتی با دوبار مشتق گرفتن و استفاده از این که k بزرگتر از دو هست و x هم بین صفر و یکه نتیجه میشه تابع محدبه ، پس مقدار مینیمم اون به ازای3/( x+y+z )به دست میاد(قضیه ی ینسن) که برابر یک سومه و مساله به راحتی حل میشه
 
آخرین ویرایش توسط مدیر

Dadgarnia

New Member
ارسال ها
1,350
لایک ها
1,127
امتیاز
0
پاسخ : ماراتن نامساوي

کلا چیزی که واضحه نامساوی همگنه پس فرض میکنیم :x+y+z=1
حالا اگه تابع f رو برابر((x/(x+k(1-x تعریف کنیم به راحتی با دوبار مشتق گرفتن و استفاده از این که k بزرگتر از دو هست و x هم بین صفر و یکه نتیجه میشه تابع محدبه ، پس مقدار مینیمم اون به ازای3/( x+y+z )به دست میاد(قضیه ی ینسن) که برابر یک سومه و مساله به راحتی حل میشه
راهتون درسته ولي اين سوال خيلي ساده تر از اينه كه بخواد با ينسن حل بشه لطفا سعي كنين از نامساوي هاي ساده تر استفاده كنين (اينو تا چند وقت پيش همه به خودم مي گفتن :4:) براي مثال توي اين سوال با يه كوشي سوال حل ميشه البته استفاده از نامساوي هايي مثل ينسن هم مهارت بالايي مي خواد.
لطفا سوال بعد رو هم بذارين.
 

REZA 73

Active Member
ارسال ها
139
لایک ها
184
امتیاز
43
پاسخ : ماراتن نامساوي

راهتون درسته ولي اين سوال خيلي ساده تر از اينه كه بخواد با ينسن حل بشه لطفا سعي كنين از نامساوي هاي ساده تر استفاده كنين (اينو تا چند وقت پيش همه به خودم مي گفتن :4:) براي مثال توي اين سوال با يه كوشي سوال حل ميشه البته استفاده از نامساوي هايي مثل ينسن هم مهارت بالايي مي خواد.
لطفا سوال بعد رو هم بذارين.
بله حق با شماست!!! دیگه وقت المپیاد از من گذشته واسه همین خیلی به سوالات جدید دسترسی ندارم پس بهتر خودتون سوال بذارید:)
 

Dadgarnia

New Member
ارسال ها
1,350
لایک ها
1,127
امتیاز
0
پاسخ : ماراتن نامساوي

سوال بعد:
براي اعداد مثبت
با شرط
ثابت كنيد:

 

M_Sharifi

راهبر ریاضی
ارسال ها
1,981
لایک ها
801
امتیاز
0
پاسخ : ماراتن نامساوي

من فرض خاصی نکردم ولی چیزی که مشهوده اگه a بزرگتر از یک باشه اون نامساوی درسته، البته میشه با روش لاگرانژ درستی این نابرابری جزئی رو به راحتی چک کرد، من با لاگرانژ بررسی کردم، ولی به احتمال زیاد یه اثبات خیلی ساده تر داره، متاسفانه به دلیل درس های زیاد دانشگاه فرصت نمیشه بیشتر از این رو مسایل فک کنم .
اون نابرابری در حالت a<1 لزوما درست نیست. مثلا
 

aras2213

New Member
ارسال ها
216
لایک ها
228
امتیاز
0
پاسخ : ماراتن نامساوي

سوال بعد:)206:)
برای اعداد حقیقی و مثبت a,b,c نشان دهید که:
.
 

AHZolfaghari

Well-Known Member
ارسال ها
935
لایک ها
1,654
امتیاز
93
پاسخ : ماراتن نامساوي

سوال بعد:)206:)
برای اعداد حقیقی و مثبت a,b,c نشان دهید که:
.
نابرابری که به واسک معروفه !!!!




---- دو نوشته به هم متصل شده است ----

سوال بعد :
a,b,c اعداد مثبت اند بطوریکه

ثابت کنید
 

REZA 73

Active Member
ارسال ها
139
لایک ها
184
امتیاز
43
پاسخ : ماراتن نامساوي

اون نابرابری در حالت a<1 لزوما درست نیست. مثلا
بله! چیزی که مشهوده اینه که حق با شماست برای حالتی که اون نامساوی برقرار نیست باید راه حلمو کامل کنم. :39:
 

aras2213

New Member
ارسال ها
216
لایک ها
228
امتیاز
0
پاسخ : ماراتن نامساوي

نابرابری که به واسک معروفه !!!!




---- دو نوشته به هم متصل شده است ----

سوال بعد :
a,b,c اعداد مثبت اند بطوریکه

ثابت کنید
.با جایگذاری در صورت سوال کافی است ثابت کنیم که :



پس اگر
، کافیست ثابت کنیم:


از طرفی
.پس با جایگذاری کافیه که ثابت کنیم:
.

پس کافیه که ثابت کنیم :
که این هم سخت نیست.

انشا... که جوب نداره:4:
 
بالا